For any given ticket in a 1000–ticket lottery, it is reasonable to believe that that ticket will lose. Hence, it is ...

Batman on November 29, 2014

Help

I don't understand why the answer goes to (A), not (B).

Replies
Create a free account to read and take part in forum discussions.

Already have an account? log in

Naz on December 5, 2014

Here we have a part to whole flaw. We know that any given ticket, i.e. any part, in a 1000-ticket lottery, i.e. in a whole, has the characteristic of it being reasonable that that ticket will lose. Thus, we will place that same characteristic for the part to the whole: "it is reasonable to believe that no ticket will win."

Answer choice (A) has the same flawed reasoning: "It is reasonable to believe for any randomly drawn playing card that it will not be an ace, so it is reasonable to believe that an ace will never be drawn."

So we are told that any randomly drawn playing card, i.e. any part, has a specific characteristic--that it is reasonable to believe that it will not be an ace. We then conclude from this that this specific characteristic for the part will also hold true for the whole: "so it is reasonable to believe that an ace will never be drawn."

Thus, answer choice (A) has the same part-to-whole flaw as in the stimulus.

Answer choice (B) is not a part-to-whole flaw.

Hope that was helpful! Please let us know if you have any other questions.

NickJFall on May 12, 2020

I had the same issue. Thanks for you the help!